nzheng
Thanks Received: 0
Forum Guests
 
Posts: 10
Joined: May 18th, 2010
 
 
 

Q17 - Researcher: Hard water contains more

by nzheng Tue May 18, 2010 12:16 pm

Hi, for this question I picked C and eliminated E because I thought it was out of scope. Could someone please explain to me why E is right? thanks.
User avatar
 
ManhattanPrepLSAT1
Thanks Received: 1909
Atticus Finch
Atticus Finch
 
Posts: 2851
Joined: October 07th, 2009
 
 
 

Re: Q17 - Researcher: Hard water contains more

by ManhattanPrepLSAT1 Tue May 18, 2010 4:28 pm

I just went through and eliminated all the answers before coming back and realizing how answer choice (E) affected the argument. This is a tough one.

The conclusion in the argument is that people who drink soft water incur a risk of heart disease, and other conditions. The evidence for this is that soft water is low in magnesium and people who suffer from heart disease have low levels of magnesium.

This argument posits a causal relationship after merely ascertaining a correlation between low magnesium and heart disease. The argument concludes that low magnesium causes the heart disease. The argument failed to consider the possibility that low magnesium was the effect of heart disease rather than the cause of heart disease. Answer choice (E) simply points that out.

(A) is irrelevant. The frequency of magnesium deficiency tells me nothing of the presumed effects of low magnesium.
(B) strengthens the argument. Relates how a magnesium deficiency could lead to heart disease.
(C) is irrelevant to the argument. There is no distinction in the argument related to age, nor would a distinction impact the conclusion.
(D) discusses how magnesium works for those who already have heart disease, but does not discuss contributing factors to developing heart disease.
(E) weakens the argument by pointing out that what the argument presumed was an effect of low magnesium could have been a cause of low magnesium instead.
 
ebrickm2
Thanks Received: 2
Forum Guests
 
Posts: 44
Joined: March 07th, 2010
 
 
 

Re: PT 40, S1, Q17 Hard water contains....

by ebrickm2 Sat Aug 21, 2010 12:52 pm

Are we to assume that since the medication lowers the magnesium levels that they are not actually at increased risk, because decreased levels of magnesium are in fact beneficial for health?

Having a low Mg level, as we are told by E, is supposed to be an effective treatment method, so it is not the case that drinking soft water will be harmful?

Am I on track?!
 
cyruswhittaker
Thanks Received: 107
Forum Guests
 
Posts: 246
Joined: August 11th, 2010
 
 
trophy
Most Thanked
trophy
First Responder
 

Re: PT 40, S1, Q17 Hard water contains....

by cyruswhittaker Wed Sep 15, 2010 9:31 pm

I don't think you're suppose to assume that.

The argument says that those who drink soft water are at an increased risk of heart disease, stroke, and hypertention, and then it gives the reason that people being treated for these conditions typically have lower levels of magnesium. So the author's reasoning seems to be that this shows that decreased magnesium causes heart disease, and since hard water also contains more magnesium than soft water, those drinking soft water are at risk (causal chain).

E simply challenges the unstated assumption that the causal assumption is correct between low magnesium and increased risk. It challenges this by saying that the low magnesium might be caused by the treatment compounds, and hence there is only a correlation between low magnesium and increased risk.
 
lisahollchang
Thanks Received: 5
Forum Guests
 
Posts: 48
Joined: August 26th, 2010
 
 
 

Re: PT 40, S1, Q17 Hard water contains....

by lisahollchang Wed Oct 20, 2010 5:05 pm

I ended up choosing E correctly for this question, but E really threw me off because it lists medications for hypertension and heart attack as having this effect. It leaves out strokes from the list, but they are mentioned in the stimulus.
User avatar
 
ManhattanPrepLSAT1
Thanks Received: 1909
Atticus Finch
Atticus Finch
 
Posts: 2851
Joined: October 07th, 2009
 
 
 

Re: PT 40, S1, Q17 Hard water contains....

by ManhattanPrepLSAT1 Thu Oct 21, 2010 1:08 pm

Think of it this way. Answer choice (E) undermines part of the conclusion. When you're asked to undermine a conclusion there are two things to consider. First, you don't need to entirely destroy the conclusion. Casting doubt on the conclusion is good enough. And second, you don't have to undermine every piece of the conclusion. Undermining part of it is good enough.

And I completely agree with cyruswhittaker's explanation above, great work!
 
gaheexlee
Thanks Received: 10
Elle Woods
Elle Woods
 
Posts: 55
Joined: May 27th, 2014
 
 
 

Re: PT 40, S1, Q17 Hard water contains....

by gaheexlee Sun Nov 16, 2014 5:24 pm

Can you also say that D is incorrect because if we choose (D) we would have to assume that no significant difference exists between magnesium supplements and magnesium found in water?
User avatar
 
maryadkins
Thanks Received: 641
Atticus Finch
Atticus Finch
 
Posts: 1261
Joined: March 23rd, 2011
 
 
 

Re: Q17 - Researcher: Hard water contains more

by maryadkins Sat Nov 22, 2014 6:01 pm

No, because magnesium is magnesium no matter what vehicle it comes in. I'm fine with that.

(D) is wrong really for the reason Matt said above.
 
contropositive
Thanks Received: 1
Atticus Finch
Atticus Finch
 
Posts: 105
Joined: February 01st, 2015
 
 
 

Re: PT 40, S1, Q17 Hard water contains....

by contropositive Wed Oct 28, 2015 4:33 pm

mattsherman Wrote:Think of it this way. Answer choice (E) undermines part of the conclusion. When you're asked to undermine a conclusion there are two things to consider. First, you don't need to entirely destroy the conclusion. Casting doubt on the conclusion is good enough. And second, you don't have to undermine every piece of the conclusion. Undermining part of it is good enough.




Hi Matt, I agree with your explanation here. But when I read this explanation it reminded me of the explanation you provided on answer choice C in
PrepTest 61 -- Section 2 LR - Q. 11, which had the same question type. In fact, when I approached question 17 in this test I had it down to C and E but I recalled the explanation you provided on that test question and I eliminated E because it did not explain "strokes."

For preptest 61 you said, "The psychologists observed three different rates. One for when no one is waiting, one for when someone is waiting, and one for when someone honks. Answer choice (C) would explain the difference in time between no one waiting and someone waiting for the parking spot. However, it would not explain the difference when someone honks the horn. Answer choice (A) however, would explain all three different times and so provides an alternative explanation to the observation in the stimulus."
User avatar
 
ohthatpatrick
Thanks Received: 3808
Atticus Finch
Atticus Finch
 
Posts: 4661
Joined: April 01st, 2011
 
 
 

Re: Q17 - Researcher: Hard water contains more

by ohthatpatrick Fri Oct 30, 2015 4:08 pm

On the question from test 61, there were several tempting answers, so we needed to closely measure which one MOST weakened.

(A), (C), and (E) are all able to offer a potential explanation for at least part of the observed phenomena. So Matt had to consider which did a better job of explaining all three parts of the observed phenomena because that's the thought process that was needed to determine which of those three was the best available answer.

In this example from test 40, (E) doesn't really have any competition, so it doesn't really matter that it's less than perfect.

Let me know if that doesn't make sense.
 
contropositive
Thanks Received: 1
Atticus Finch
Atticus Finch
 
Posts: 105
Joined: February 01st, 2015
 
 
 

Re: Q17 - Researcher: Hard water contains more

by contropositive Fri Oct 30, 2015 7:30 pm

ohthatpatrick Wrote:On the question from test 61, there were several tempting answers, so we needed to closely measure which one MOST weakened.

(A), (C), and (E) are all able to offer a potential explanation for at least part of the observed phenomena. So Matt had to consider which did a better job of explaining all three parts of the observed phenomena because that's the thought process that was needed to determine which of those three was the best available answer.

In this example from test 40, (E) doesn't really have any competition, so it doesn't really matter that it's less than perfect.

Let me know if that doesn't make sense.



That makes sense, thank you. The way I have learned to attack all these questions that have "most" in the question stem is to just look at it as there is only 1 answer, not the "most" better. But I will apply your rule next time.

I just want to be clear, so C is wrong because there are no distinctions made in the argument about peoples' age. If I come across a similar argument and question type, do I have to make sure that "age" has been discussed in the argument?
User avatar
 
ohthatpatrick
Thanks Received: 3808
Atticus Finch
Atticus Finch
 
Posts: 4661
Joined: April 01st, 2011
 
 
 

Re: Q17 - Researcher: Hard water contains more

by ohthatpatrick Thu Nov 05, 2015 4:37 pm

(C) isn’t wrong because ‘age’ was never mentioned, or else (E) would also be wrong because ‘compounds used to treat hypertension’ were never mentioned.

New information is fine. In fact, it’s mandatory. We’re looking for a new idea that changes our thinking on this argument.

So you’re just measuring whether (C) changes your thinking.

I would be tempted by (C) as an alternate explanation for why people being treated for heart disease / stroke / hypertension have low magnesium in their blood.

I might think, “Oh, it’s not that low magnesium causes these conditions. It’s that being OLD separately causes you to have these conditions and to have low magnesium in your blood.”

In order for (C) to have that function, I have to add in an assumption:
- old people are more likely to have these three conditions than other people

That’s reasonable with outside knowledge, but pretty off the page here. Plus, (C) doesn’t say “OLD people have a lower ability to metabolize magnesium” … it says “a 10 year old has worse magnesium metabolization than a 5 year old … a 30 year old’s is worse than a 20 year old’s … etc.”

It’s a little messy because it applies to an entire life span. It would be hard to know at what point in life you “tend to have lower levels of Mg in your blood” than the rest of the population and hard to know if that age and onward incurs a higher risk of the three conditions.

(E), meanwhile, requires no extra story. “Oh, it’s not that low Mg causes these conditions. It’s that when you have these conditions, you’re taking medicine that lowers your Mg.”
 
contropositive
Thanks Received: 1
Atticus Finch
Atticus Finch
 
Posts: 105
Joined: February 01st, 2015
 
 
 

Re: Q17 - Researcher: Hard water contains more

by contropositive Thu Nov 05, 2015 6:23 pm

makes a lot of sense now thank you!
 
haeeunjee
Thanks Received: 15
Jackie Chiles
Jackie Chiles
 
Posts: 37
Joined: May 05th, 2016
 
 
 

Re: Q17 - Researcher: Hard water contains more

by haeeunjee Tue Jan 10, 2017 3:41 am

ManhattanPrepLSAT1 Wrote:(D) discusses how magnesium works for those who already have heart disease, but does not discuss contributing factors to developing heart disease.


But (E) is also about people who already have heart disease and are being treated with "compounds."

I could make the case for (D) by saying that people who are being treated for heart disease avoid taking mag supplements because they inhibit the effectiveness of the heart medicine. Therefore, on average, people with heart disease might exhibit lower mag levels than someone w/o heart disease, and this is why there's a correlation, weakening the original argument.

I think (D) is wrong because we have to make additional assumptions: 1- That people with heart disease don't have average levels of mag to begin with (if they don't need to take mag supplements in the first place, then they'll just have the same levels of mag as the rest even without taking supplements), 2- That abstaining from mag supplements leads to lower-than-average mag levels (what if the body just generates more, but it takes time?)

I was also nitpicking at the language in (D) and (E). "The ingestion of mag supplements inhibits the effectiveness of many medicines" vs "Compounds commonly used to treat hypertension..." Does "commonly" have more force than "many"? What I mean is, is "commonly" on the same force level as "generally"/"usually"/"most of the time"? If it is, then it's stronger than "many," which also might help make (E) a more forceful Weakener. Not sure if this helps particularly on this problem, but maybe for future questions, this would be helpful.